Physics > Work Energy and Power > 2.0 Work done by a constant force

  Work Energy and Power
    1.0 Introduction
    2.0 Work done by a constant force
    3.0 Spring Force
    4.0 Conservative & Non-conservative forces
    5.0 Kinetic Energy $(K)$
    6.0 Potential energy $\left( {\Delta U} \right)$
    7.0 Work energy theorem
    8.0 Power
    9.0 Types of equilibrium
    10.0 Work done by a distributed mass

2.2 Work done by a variable force
Let us consider a variable force $F$ acting on a body. The force $F$ is a function of $x$ as shown by the graphical relation.

Work done on a body by a variable force can be written as, $$ W = \int\limits_{{x_1}}^{{x_2}} {\overrightarrow F (x).d\overrightarrow x } $$

The area marked by brown color is the work done by a variable force in displacing a body by a distance $dx$.
The area marked by sky blue color is the work done by a variable force in displacing the body from $x_1$ to $x_2$.

Question 5. A variable force $F=3x^2+2x$ is applied on a body. Find the work by the force in displacing the body along the $x$ axis by a distance $2m$ from the origin.

Solution: Work done is given as $$\begin{equation} \begin{aligned} W = \int {\overrightarrow F .d\overrightarrow x } \\ W = \int\limits_0^2 {\left( {3{x^2} + 2x} \right)dx} \\ W = \left[ {{x^3} + {x^2}} \right]_0^2 \\ W = 12J \\\end{aligned} \end{equation} $$

Question 6. A force $\overrightarrow F = \left( {3{x^2}\widehat i + 2y\widehat j + 2\widehat k} \right)$ displaces a body from position $(2,3,4)$ to $(1,2,3)$. Find the work done.

Solution: Small displacement in 3D space is given as, $d\overrightarrow r = \left( {dx\widehat i + dy\widehat j + dz\widehat k} \right)$.

Therefore the work done is, $$\begin{equation} \begin{aligned} W = \int\limits_{(2,3,4)}^{(1,2,3)} {\overrightarrow F .d\overrightarrow r } \\ W = \int\limits_{(2,3,4)}^{(1,2,3)} {\left( {3{x^2}\widehat i + 2y\widehat j + 2\widehat k} \right).\left( {dx\widehat i + dy\widehat y + dz\widehat k} \right)} \\ W = \int\limits_{(2,3,4)}^{(1,2,3)} {\left( {3{x^2}dx + 2ydy + 2dz} \right)} \\ W = \left[ {{x^3}} \right]_2^1 + \left[ {{y^2}} \right]_3^2 + \left[ {2z} \right]_4^3 \\ W = (1 - 8) + (2 - 9) + (6- 8) \\ W = - 16J \\\end{aligned} \end{equation} $$

Note: Work done is negative because the direction of force and displacement is not same.
Improve your JEE MAINS score
10 Mock Test
Increase JEE score
by 20 marks
Detailed Explanation results in better understanding
Exclusively for
JEE MAINS and ADVANCED
9 out of 10 got
selected in JEE MAINS
Lets start preparing
DIFFICULTY IN UNDERSTANDING CONCEPTS?
TAKE HELP FROM THINKMERIT DETAILED EXPLANATION..!!!
9 OUT OF 10 STUDENTS UNDERSTOOD